A number raise to it self infinitely

  • Thread starter bigjoe5263
  • Start date
  • Tags
    Self
Does anyone have any ideas on how to find x when x^y = y^x for x<>y, x<e?In summary, the conversation discusses the problem of finding the value of X in the equation x^{(x^{(x^{(x^...}})} = 3. It is noted that this equation does not converge for all values of x, and the upper bound for a solution is between 2 and 3. Additionally, it is mentioned that the function g(x) = x^{1/x} reaches a maximum at x = e, and hence for N > e, the equation x^{(x^{(x^{(x^...}})} = N has no solution.
  • #1
bigjoe5263
19
0
hi I have encountered this...

.
Code:
                                     . 
                                   X
                                X
                              X
                           X    =   3
I tried to solve it using properties of logarithims however I always end up with the initial equation... how will I find the value of X? ( x is raised to it self endlessly ).

:confused:
.
 
Last edited:
Physics news on Phys.org
  • #2
You have to parenthesize that exponential tower or it's ambiguous in meaning. 3^(3^3) is not equal to (3^3)^3. And as it's infinite, you need to express the meaning in terms of an infinite sequence. So start with a_0=x. Is a_n=x^(a_(n-1)) or is a_n=(a_(n-1))^x?
 
Last edited:
  • #3
The generally understood meaning of the left hand side of that equation is the infinite power tower function

[tex]f(x) = x^{(x^{(x^{(x^...[/tex]

or simply, work "downwards" from "infinity".

The "trick" solution to f(x) = N is to see that this implies x^f(x) = N hence x^N = N when a solution exists, giving x = N^(1/N).

The problem is that there is an upper bound on N for a solution to exist. I'll leave it to the OP to find that bound and hence deduce that f(x) = 3 has no solution.
 
  • #4
If [itex]x^{(x^{(x^{(x^...}}} = 3[/itex], Then
x3= [tex]x^[{(x^{(x^{(x^...]}}}}= x^{(x^{(x^{(x^...}}}[/tex]. What is that equal to?

Curios3141, I don't see why that "has no solution". Am I missing something?
 
  • #5
Halls, try calculating [tex]f(x)=x^{x^{x^\cdots}}[/tex] when [tex]x=3^{1/3}[/tex]. It does not converge to 3. It does not converge, period. The upper bound on N is between 2 and 3 and is a very ubiquitous number.
 
  • #6
How are you calculating that? If, for example, you calculate 31/3, store it in "x" in a calculator, do x^x, then "ans"^x, repeatedly, it does not converge.

But that is NOT [tex]f(x)=x^{x^{x^\cdots}}[/tex]! To calculate that you need to repeatedly do x^"ans" and that does converge, slowly, to 3.
 
  • #7
I misspoke. It does converge, just not to the value you expect. Let

[tex] f_1(x) = x[/tex]
[tex] f_k(x) = x^{f_{k-1}(x)}[/tex]

The desired function [itex]f(x)[/itex] is

[tex] f(x) = x^{x^{x^{\cdots^x}} = \lim_{k\to\infty}f_k(x) [/tex]

Now further define

[tex]g(x) = x^{1/x}[/tex]

THe question at hand is [itex]f(g(x))=x[/itex]? For [itex]x \in [ 1/e,e ] [/itex], [itex]f(g(x))=x[/itex]. The convergence is very slow as x approaches 1/e or e.

Things get a bit interesting when x>e. For example, [itex]f(g(3)) = 2.478053\cdots[/itex] rather than 3. What's going on here? Note that [itex]g(3) = 1.44225\cdots[/itex]. However, [itex]g(2.478053\cdots) = 1.44225\cdots[/itex] also. The inverse of [itex]g(x)[/itex] has two branches, with a branch point at [tex]e^{1/e}[/tex]. [itex]f(g(x))[/itex] converges to [tex]g^{-1}(g(x))[/tex], where [tex]g^{-1}[/tex] refers to the lower branch.

Things get even more interesting when x<1/e. The sequence [itex]\{f_k(x)\}[/itex] doesn't converge. Instead, the sequence toggles between a pair of values as [itex]k\to\infty[/itex], one close to zero and the other close to one.
 
  • #8
I'm a bit confused as to what converges, "converge, just not to the value you expect". The original problem was to solve
[tex\ x^{x^x^{\cdot\cdot\cdot}}= 3[/tex].
IF the sequence [itex]a_1= x^x[/itex], [itex]a_2= x^{a_1}, ... converges to 3, then x must be equal to 31/3.
 
  • #9
HallsofIvy said:
I'm a bit confused as to what converges, "converge, just not to the value you expect". The original problem was to solve
[tex\ x^{x^x^{\cdot\cdot\cdot}}= 3[/tex].
IF the sequence [itex]a_1= x^x[/itex], [itex]a_2= x^{a_1}, ... converges to 3, then x must be equal to 31/3.

That's true. But the converse (if x=3^(1/3) then the sequence converges to 3) isn't true. The sequence does converge to a limit L that you can find by solving log(L)/L=log(3)/3 (not the L=3 root, the other root L=2.47805...).
 
  • #10
I am going to have to meditate upon this!
 
  • #11
HallsofIvy said:
IF the sequence [itex]a_1= x^x[/itex], [itex]a_2= x^{a_1}, ...[/itex] converges to 3, then x must be equal to 31/3.

The problem is that no x exists such that the sequence [itex]\{a_k(x)\}[/itex] converges to 3.
 
  • #12
HallsofIvy said:
I am going to have to meditate upon this!

Halls, it's simple. The radius of convergence of the hyperpower function [tex]f(x) = ^{\infty}x[/tex] (using the tetration notation) is [tex]({(1/e)}^e, e^{(1/e)})[/tex]

For x > e^(1/e), the function does not coverge.

For a value like 3^(1/3), the function *does* converge. But it does not converge to 3.

The function [tex]g(x) = x^{(\frac{1}{x})}[/tex] reaches a maximum at x = e. In other words, you cannot find a value of x that makes g(x) > e^(1/e), so g(f(x)) will always converge for any real positive x > e.

So for N > e, f([N^(1/N)]) will never equal N, even though convergence will hold. Hence f(x) = N has no solution for x > e.

Hope that clarifies things. I haven't explored the lower bound this carefully, so I shan't comment on it.
 
Last edited:
  • #13
Yes, thanks. I showed that "if the sequence of powers converges to 3, then x= 31/3. The converse, "if x= 31/3, then the sequence converges to 3" is not true.
 
  • #14
HallsofIvy said:
Yes, thanks. I showed that "if the sequence of powers converges to 3, then x= 31/3. The converse, "if x= 31/3, then the sequence converges to 3" is not true.

Of course, the first condition (convergence to 3) will never actually be met.
 
  • #15
D_H's f(g(x)) appears to provide solutions for x^y = y^x, x<>y, x>e.
For instance y=f(g(3)) is the other solution for 3^y = y^3, besides the obvious y=3.
When x=e, the only solution is y=e.
However, such solutions also exist when 1 < x < e, eg 2^4 = 4^2, but f(g(x)) does not help then, eg f(g(2)) = 2.
 
Last edited:

Related to A number raise to it self infinitely

1. What does it mean to raise a number to itself infinitely?

Raising a number to itself infinitely means taking a number and continuously multiplying it by itself an infinite number of times. This results in an infinitely increasing number.

2. Can a number raised to itself infinitely ever be equal to zero?

No, a number raised to itself infinitely will never be equal to zero. This is because the number will continue to increase infinitely and will never reach zero.

3. Is it possible for a number raised to itself infinitely to be negative?

Yes, it is possible for a number raised to itself infinitely to be negative. This can happen if the original number is negative and the exponent is an odd number. However, if the exponent is an even number, the result will always be positive.

4. How does raising a number to itself infinitely differ from raising it to a finite exponent?

Raising a number to itself infinitely is different from raising it to a finite exponent because the result will be infinitely increasing instead of a specific number. Also, when raising to a finite exponent, the number will eventually reach a maximum value, whereas with infinite exponentiation, there is no maximum value.

5. Can any number be raised to itself infinitely?

Yes, any number can be raised to itself infinitely. This includes both positive and negative numbers, as well as fractions and irrational numbers.

Similar threads

  • Precalculus Mathematics Homework Help
Replies
18
Views
1K
  • Precalculus Mathematics Homework Help
Replies
2
Views
902
  • Precalculus Mathematics Homework Help
Replies
7
Views
3K
  • Precalculus Mathematics Homework Help
Replies
2
Views
430
  • Precalculus Mathematics Homework Help
Replies
21
Views
1K
  • Precalculus Mathematics Homework Help
Replies
8
Views
573
  • Precalculus Mathematics Homework Help
Replies
4
Views
1K
  • Precalculus Mathematics Homework Help
Replies
11
Views
1K
  • Precalculus Mathematics Homework Help
Replies
17
Views
1K
  • Precalculus Mathematics Homework Help
Replies
10
Views
938
Back
Top